LSAT and Law School Admissions Forum

Get expert LSAT preparation and law school admissions advice from PowerScore Test Preparation.

 Administrator
PowerScore Staff
  • PowerScore Staff
  • Posts: 8917
  • Joined: Feb 02, 2011
|
#25111
Complete Question Explanation
(The complete setup for this game can be found here: lsat/viewtopic.php?t=4009)

The correct answer choice is (C)

If L is placed on a higher shelf than H, then either Template #2 or #3 is in effect. And, because O must be placed on a higher shelf than L, we can deduce that O is on the top shelf, L is on the middle shelf, and H is on the bottom shelf. The remaining two books—G and K—have several options, although at least one of the two must be on the top shelf with O:
powerscore_M12_T4_O2011_LG_explanations_game_4_#23_diagram_1.png
Because H and M must always be placed on the same shelf, answer choice (C) is correct.

Note how each of the incorrect answers includes G or K, which is where the uncertainty in this question appears. In a Must Be True question (such as this one), avoid uncertainty in the correct answer. In a Could Be True question, look for the uncertainty in the correct answer.
You do not have the required permissions to view the files attached to this post.

Get the most out of your LSAT Prep Plus subscription.

Analyze and track your performance with our Testing and Analytics Package.